9
$\begingroup$

For some research work, I need to know the classification of elements of finite order of $\mathrm{PGL}(n,\mathbb{Q})$, up to conjugation.

Since I essentially need $n\le 4$, I think that I can show it by hand, using cyclotomic extensions and Galois theory, but is there some work in the literature on this?

EDIT: Looking at the possible orders is essentially trivial in $\mathrm{GL}(n,\mathbb{Q})$, by just looking at the cyclotomic polynomials. The conjugacy classes require a little more work but are easy exercises, at least in low dimension. For $\mathrm{PGL}(n,\mathbb{Q})$, the case of orders prime to $n$ follows essentially from the case of $\mathrm{GL}(n,\mathbb{Q})$, the orders are more interesting.

$\endgroup$
15
  • 1
    $\begingroup$ there seems to be a lot known about finite subgroups of $GL_n(\mathbb{Q})$, cf. e.g. references in ams.org/journals/proc/1997-125-12/S0002-9939-97-04283-4/… E.g. each of them is conjugate to one in $GL_n(\mathbb{Z})$. $\endgroup$ Jun 1, 2014 at 10:32
  • $\begingroup$ Isn't this just a question about rational canonical form? The question is certainly easier for elements of finite order in ${\rm GL}(n,\mathbb{Q}),$ but even there, calculating the maximum possible order of an element of finite order in ${\rm GL}(n,\mathbb{Q})$ is quite subtle. $\endgroup$ Jun 1, 2014 at 11:01
  • $\begingroup$ @GeoffRobinson: what's so subtle in $GL_n(Q)$? you need to list $L_n$, the set of all cyclotomic polynomials of degree $\le n$, then from $L_n$ you list $L'_n$, the set of all subsets of $L_n$ whose sum of degrees is $\le n$. For each $\{\Phi_{n_1},\dots,\Phi_{n_k}\}$ in $L'_n$, you get an element of order lcm$(n_1,\dots,n_k\}$ in $GL_n(Q)$. Of course I don't claim it's algorithmically efficient when $n$ is very large. $\endgroup$
    – YCor
    Jun 1, 2014 at 11:26
  • $\begingroup$ @Yves Cornulier: I just meant that that lcm is not so easy to explicitly evaluate, although theoretically, as you say, it is a formality. $\endgroup$ Jun 1, 2014 at 11:30
  • $\begingroup$ @Jérémy: my guess is that a natural approach would be to start up to conjugation in $PGL_n(C)$, and then to understand when two elements in $PGL_n(Q)$ are conjugate over $C$ are conjugate over $Q$: since this problem is trivial in $GL_n$, one can expect it to we well encoded (in Galois cohomology?) in $PGL_n$. $\endgroup$
    – YCor
    Jun 1, 2014 at 11:37

4 Answers 4

4
$\begingroup$

I figured I'd write up the elementary observations here, since no one else has: If $g^k = \mathrm{Id}$ in $PGL_n$, then $g^k = a \mathrm{Id}$ in $GL_n$ for some nonzero $a$. So the minimal polynomial of $g$ divides $x^k-a$. Let $p_1 p_2 \cdots p_r$ be the factorization of $x^k-a$ over $\mathbb{Q}$. We can use rational canonical form to write down a $\deg p_i \times \deg p_i$ matrix $g_i$ (over $\mathbb{Q}$) with characteristic polynomial $p_i$. Then, for any nonnnegative integers $a_i$ such that $\sum a_i \deg p_i = n$, we can take the block diagonal matrix whose entries are $a_i$ copies of $g_i$. Conversely, if $g^k = a$, we can break $g$ into blocks according to the irreducible factors of $x^k-a$.

So, what remains is to analyze the degrees of the $p_i$. Let $K$ be the splitting field of $x^k-a$ over $\mathbb{Q}$ and let $G$ be the Galois group. Write $\zeta$ for a primitive $k$-th root of $1$ and $\alpha$ for a chosen $k$-th root of $a$ inside $K$. Then every element of $G$ is of the form $\zeta^i \alpha \mapsto \zeta^{ui+v}$ for $u \in (\mathbb{Z}/k)^{\times}$ and $v \in \mathbb{Z}/k$. So $G$ is a subgroup of $(\mathbb{Z}/k)^{\times} \ltimes (\mathbb{Z}/k)$. Also, $G$ surjects onto $\mathrm{Gal}(\mathbb{Q}(\zeta)/\mathbb{Q})$, so, for every $u \in \mathbb{Z}/k^{\times}$, the group $G$ contains an element of the form $i \mapsto ui+v$. The problem is to describe the orbits of such a group on $\mathbb{Z}/k$.

For example, when $k=4$, then $G$ is a subgroup of $\{ \pm 1 \} \ltimes (\mathbb{Z}/4)$. If it isn't the whole group (in which case $x^4-a$ is irreducible), then it is either the group generated by $i \mapsto -i$ (in which case $x^4-a$ factors as $(\mbox{linear}) (\mbox{linear})(\mbox{quadratic})$, like $x^4-1$), or the group generated by $i \mapsto -i+1$ (in which case $x^4-a$ factors as $(\mbox{quadratic}) (\mbox{quadratic})$, like $x^4+4=(x^2+2x+2)(x^2-2x+2)$), or else $\{ \pm 1 \} \times 2 \mathbb{Z}/4$, in which case (in which case $x^4-a$ factors as $(\mbox{quadratic}) (\mbox{quadratic})$, like $x^4-4$).

At this point, it isn't clear what to do next, and the question is also a bit unfocused. Here are some (in my opinion) natural questions:

  • Is is true that $x^k-a$ always has a factor of degree $\geq \phi(k)$? UPDATE: No. $x^8-16 = (x^2-2)(x^2+2)(x^2-2x+2)(x^2+2x+2)$, and $\phi(8) = 4$.

  • For fixed $k$, what is the smallest $n$ for which $PGL_n(\mathbb{Q})$ has an element of order $k$? It is NOT $\phi(k)$: We can build elements of order $15$ in $GL_6$ as the direct sum of elements of orders $3$ and $5$ in $GL_2$ and $GL_4$, even though $\phi(15) = 8$.

  • I don't have an example of a case where $PGL_n(\mathbb{Q})$ has an element of order $k$ but $GL_n(\mathbb{Q})$ doesn't. I imagine such a thing exists, but it would be good to have an example.

$\endgroup$
1
  • 1
    $\begingroup$ For your Q2, I think it's sum over $p$ dividing $k$ of $\phi(p^r)$, where $r$ is the highest power of $p$ dividing $k$, except if $p=2$ and $r=1$ in which case you can ignore that term. This is because you have to include each prime power, and multiplying them multiplies their $\phi$, but adding is always less than multiplying when you're multiplying things at least $2$. $\endgroup$
    – Will Sawin
    Aug 31, 2014 at 20:12
1
$\begingroup$

Let $\lambda^k=a$. Consider how the Galois group of the splitting field of the minimal polynomial of $\lambda$ acts on the roots. As David Speyer points out, this is a subgroup of $(\mathbb Z/k)^\times \ltimes (\mathbb Z/k)$. If this subgroup has a nontrivial intersection with $ (\mathbb Z/k)$, then there is a $l$ dividing $k$ sucht hat for any root $\lambda$ of the minimal polynomial, and $l$th root of unity $\mu$, $\mu \lambda$ is also a root.

So the degree of the minimal polynomial of $\lambda$ is $l$ times the degree of the minimal polynomial of $\lambda^l$. Now we may reduce to the case where the Galois group has trivial intersection with $\mathbb Z/k$.

Observation 1: In this case the Galois group is abelian, so the size of the orbit is the size of the Galois group, so the size of the orbit is at least $\phi$ of the order of the element. So no new orders of elements of $PGL_n(\mathbb Q)$ appear that aren't orders of elements of $GL_n(\mathbb Q)$.

Observation 2: If $k$ is odd, then there is some element of the form $x \to 2x+b$ in the Galois group. This has the unique fixed point $x=-b$. Because the Galois group is abelian, every other element fixes this point. So the $k$th roots of $a$ are some rational number times the $k$th roots of unity.

Full classification: Let $m \in (\mathbb Z/k)^\times$ be an element such that $m-1 \in 2 (\mathbb Z/k)^\times$. Then there is an element $x \to mx+b$ in the Galois group. If $b$ is even, then this element has two fixed points, the solutions to $(m-1)x+b=0$, so every other element either fixes or reverses those points. What this means is that there is a quadratic extension of $\mathbb Q$ fixed by the index $2$ subgroup of the Galois group that fixes those two points, and that those two points are of the form $q \sqrt{D}$ where $q$ is rational and $\sqrt{D}$ generates that quadratic extension. One can easily classify all sets of roots of this type.

I see my argument in the case $b$ odd doesn't actually work so I don't have a full classification.

$\endgroup$
2
  • $\begingroup$ You can avoid the case $b$ odd: Replacing $x^n-a$ by $x^{2n}-a^2$, we may assume that our equation has a real root. Choose that real root to call $\alpha$, then complex conjugation acts by $\zeta^x \alpha \mapsto \zeta{-x} \alpha$, and that puts you in the case $b$ even. At this point, I think I know everything that can happen when factoring $x^n-a$ (including fun things like $x^{10} - 5^5$) but I'm having trouble organizing it into a succinct answer. If you can, please go ahead. $\endgroup$ Sep 1, 2014 at 20:45
  • $\begingroup$ PS: There is not always an orbit of size $\geq \phi(n)$. Check out $x^8-16$. $\endgroup$ Sep 1, 2014 at 20:46
0
$\begingroup$

It seems to me that the question is equivalent to classifying up to conjugacy in ${\rm GL}(n,\mathbb{Q}),$ those matrices $M$ which satisfy $M^{d} = \frac{u}{v}I,$ where $d$ is a positive integer, $u$ is a positive integer, $v$ is anon-zero integer, ${\rm gcd}(u,v) = 1$ and neither $u$ nor $v$ is divisible by the $d$-th power of any prime, and furthermore, no lower power of $M$ is scalar. Then it becomes a question of determining what the possible rational canonical forms for $M$ can be. Maschke's theorem still goes through in this situation, suitably interpreted, and the minimum polynomial of $M$ is multiplicity free. The possible factors for the characteristic polynomial of such an $M$ are easy to determine.

$\endgroup$
1
  • $\begingroup$ Yes, this is the natural start. By looking at the determinant you get $\det(M)^d=(u/v)^m$. If $m$ and $d$ are prime, you can then reduce to the case of elements of finite order of $\mathrm{GL}(n,\mathbb{Q})$. The other cases are more interesting. $\endgroup$ Jun 1, 2014 at 16:15
-1
$\begingroup$

See https://math.temple.edu/~lorenz/papers/sizes/sizes.pdf

Look for The Minkowski sequence

$\endgroup$
1
  • $\begingroup$ Thanks, but it seems that the text that you mention only gives orders of finite subgroups of $\mathrm{GL}(n,\mathbb{Q})$, which is essentially trivial for cyclic groups. $\endgroup$ Jun 1, 2014 at 16:20

Your Answer

By clicking “Post Your Answer”, you agree to our terms of service and acknowledge you have read our privacy policy.

Not the answer you're looking for? Browse other questions tagged or ask your own question.